Pistón adiabático: ¿por qué el argumento de Callen es defectuoso?

Esta pregunta está relacionada con ésta . Le pediría que lea esa pregunta y mi respuesta a la pregunta misma antes de responder a esta.

El problema es el siguiente. En su libro Termodinámica , apéndice C, Callen introduce el problema del pistón adiabático : encontrar el estado de equilibrio de un sistema aislado con una pared interna adiabática móvil.

El argumento de Callen es el siguiente:


El volumen total es fijo:

V = V 1 + V 2

De modo que

d V 1 = d V 2
Y el pistón es adiabático, es decir, no permite intercambio de calor, por lo que:

d tu 1 , 2 = PAGS 1 , 2 d V 1 , 2

La entropía total del sistema es

S ( tu , V ) = S 1 ( tu 1 , V 1 ) + S 2 ( tu 2 , V 2 )

por eso

d S = d S 1 + d S 2 = d tu 1 T 1 + PAGS 1 T 1 d V 1 + d tu 2 T 2 + PAGS 2 T 2 d V 2

Ya que d tu 1 , 2 = PAGS 1 , 2 d V 1 , 2 , vemos eso d S se desvanece idénticamente, de modo que no podemos decir nada sobre PAGS 1 , 2 y T 1 , 2 : el principio de máxima entropía no es concluyente.

Por otro lado, está la conservación de la energía. requerimos eso d tu = 0 ya que nuestro sistema está aislado del medio ambiente, por lo tanto

d tu 1 + d tu 2 = 0 PAGS 1 d V 1 + PAGS 2 d V 2 = 0

Pero V = V 1 + V 2 y V es fijo, por lo que d V 1 = d V 2 y obtenemos

PAGS 1 = PAGS 2


Sé que esta conclusión es correcta , es decir, es una condición necesaria para el equilibrio termodinámico. Por otro lado, en el artículo Termodinámica de sistemas con restricciones adiabáticas internas: evolución temporal del pistón adiabático de C. Gruber (1998) el autor dice:

Luego se observó que el argumento de Callen, que fue repetido por Leff, no podía ser correcto ya que la condición de equilibrio se derivó de la primera ley, en lugar de la segunda ley.

Mi pregunta es: ¿Qué quiere decir el autor de este artículo? ¿Por qué debería ser defectuoso el argumento de Callen?

Hago hincapié en que no quiero una prueba alternativa, sino solo una explicación de lo que está mal en el argumento de Callen, posiblemente usando solo la termodinámica y no la teoría cinética o la dinámica de fluidos.


Actualizar

Una "respuesta" a esta pregunta está contenida en el artículo A Thermodynamic Consideration of Mechanical Equilibrium in the Presence of Thermally Insulating Barriers de AE ​​Curzon (1968), que se menciona explícitamente en el artículo que cito en mi pregunta. El problema es que para mí no está claro por qué el argumento de Curzon debería ser el "correcto" mientras que el de Callen es el "incorrecto"...

Este es el argumento de Curzon:

El sistema está aislado y su volumen es fijo, de modo que

(1) d tu 1 + d tu 2 = 0
(2) d V 1 + d V 2 = 0
Si asumimos que la pared que separa los dos sistemas no permite el intercambio de materia, tenemos
(3) T 1 d S 1 = d tu 1 + PAGS 1 d V 1
(4) T 2 d S 2 = d tu 2 + PAGS 2 d V 2
La entropía de un sistema aislado con volumen fijo en equilibrio es máxima:
(5) d S = d S 1 + d S 2 = 0
De 1 , 2 y 4 obtenemos
(6) T 2 d S 2 = d tu 1 PAGS 2 d V 1
Sustituyendo 6 y 3 en 5 , obtenemos
(7) d S = ( 1 T 1 1 T 2 ) d tu 1 + ( PAGS 1 T 1 PAGS 2 T 2 ) d V 1 = 0
Ahora, dado que el pistón es adiabático, tenemos para los dos subsistemas
(8) d S 1 , 2 0
(esto es una consecuencia de la desigualdad de Clausius). ecuaciones 5 y 8 solo puede ser consistente si
(9) d S 1 = d S 2 = 0
De 3 y 9 obtenemos
(10) d tu 1 = PAGS 1 d V 1
Sustituyendo 10 en 7 finalmente obtenemos
(11) ( PAGS 1 PAGS 2 ) d V 1 T 2 = 0
Siendo d V 1 arbitrario, concluimos que
PAGS 1 = PAGS 2

No estoy completamente convencido de que esto pueda considerarse la prueba "correcta" de que PAGS 1 = PAGS 2 es una condición necesaria para el equilibrio, mientras que la de Callen es incorrecta. Curzon en realidad aborda el problema de las derivaciones análogas de muchos autores (por ejemplo, Kubo), es decir, el hecho de que afirman que la condición PAGS 1 = PAGS 2 puede derivarse sólo si suponemos T 1 = T 2 . ¡ Pero Callen no hace tal suposición! De hecho, su argumento es muy similar al de Curzon y me parece que el argumento de Curzon es todo un truco matemático.

De hecho, de 4 y 9 obtendríamos d tu 2 = PAGS 2 d V 2 (análogos a 1 ): esto, junto con 1 , 2 y 10 , nos daría el argumento de Callen de nuevo!

Entonces, en conclusión, para mí parece que el argumento de Curzon es bastante similar al de Callen: por lo tanto, no está claro qué quiso decir Gruber cuando escribió esa oración, y considero que la pregunta aún está abierta.

"Incorrecto" parece una palabra muy extraña para usar aquí.
@dmckee "Incorrecto" o "defectuoso" si lo prefiere. Se ha señalado en más de un artículo que, aunque la conclusión es correcta, el argumento no lo es. Pero nadie explica claramente por qué.

Respuestas (5)

Cito del resumen de uno de los artículos que el OP vinculó en la pregunta anterior a esta

El principio de aumento de entropía se utiliza para obtener la condición de equilibrio mecánico en un sistema aislado dividido en dos partes por un pistón sin fricción y sin peso que está hecho de un material perfectamente aislante térmico. El resultado enfatiza que el principio puede usarse para obtener la condición de equilibrio mecánico sin la suposición , frecuentemente hecha en los libros de texto, de que el equilibrio mecánico está acompañado por el equilibrio térmico .

Parece que aquí se establece explícitamente que el "defecto" de Callen es esta suposición. Consulte también esta pregunta de física.SE para obtener una explicación de la diferencia entre el equilibrio mecánico y térmico en una de las respuestas.

Para resumir con mis propias palabras: PAGS 1 = PAGS 2 solo se asegura de que esté en un estado en el que nada se mueva / acelere (es decir, que esté en equilibrio mecánico), lo que obviamente sería el caso si PAGS 1 PAGS 2 (por lo tanto, es una condición necesaria, como dice el OP). Sin embargo, esto no es suficiente para concluir que estás en equilibrio termodinámico, porque debido a las fluctuaciones térmicas siempre obtienes PAGS 1 PAGS 2 parte del tiempo Ahora podría darse el caso de que el ( PAGS 1 = PAGS 2 ) -estado era metaestable y se aleja del equilibrio mecánico inducido por las pequeñas fluctuaciones. Por lo que puedo decir de los artículos vinculados en la otra pregunta, ese no es el caso, entonces PAGS 1 = PAGS 2 resulta ser la condición correcta, pero Callen no demostró rigurosamente que la "fuerza restauradora" que acompaña a las fluctuaciones conduce de nuevo al equilibrio mecánico.

No estoy 100% seguro si esto responde a la pregunta, ni si es correcto. En particular, personalmente creo que el argumento de Callen puede completarse fácilmente al señalar que el equilibrio mecánico PAGS 1 = PAGS 2 es único _ Quiero decir, si su sistema deja el equilibrio mecánico debido a las fluctuaciones, tiene que ir a alguna parte. Y no hay un segundo equilibrio mecánico al que pueda ir. es decir, necesario suficiente ya que también es único.


Actualizar

Esto es para abordar la actualización de la pregunta (v6).

Además de la pregunta original, la actualización contiene un resumen del argumento que se supone que es correcto, lo que arroja el mismo resultado que el argumento de Callen. El OP luego señala que los argumentos parecen ser los mismos.

No creo que este sea el caso. Como indicó anteriormente el OP Callen solo usa la primera ley, mientras que para una prueba correcta del equilibrio se debe emplear la segunda ley. La razón de esto es lo que traté de señalar anteriormente en mi respuesta original. La primera ley es un mero enunciado de conservación de la energía. No puede extraer información sobre el equilibrio termodinámico simplemente imponiendo las restricciones de su sistema. Eso solo muestra que se logra el equilibrio mecánico, que es una condición necesaria pero no suficiente para el equilibrio termodinámico (como OP ya afirmó repetidamente).

La segunda ley establece la suficiencia. ¿Por qué? Porque es una declaración sobre la naturaleza del equilibrio termodinámico en sí mismo, en particular te dice cómo se comportan las fluctuaciones alrededor del punto de equilibrio y que volverás a tu estado original bajo perturbaciones de fluctuación, para decirlo desde el punto de vista de la mecánica estadística.

Una de las dudas del OP es

En efecto, de (4) y (9) obtendríamos d tu 2 = PAGS 2 d V 2 (análogos a (1)): ¡esto, junto con (1), (2) y (10), nos daría el argumento de Callen nuevamente!

Por supuesto, obtendrá las mismas relaciones que con la primera ley si intercala las relaciones obtenidas con el argumento de la segunda ley, porque el equilibrio mecánico resulta ser un equilibrio térmico en la situación considerada. Sin embargo, no veo cómo esta coincidencia significa que son el mismo argumento, ya que parten de puntos completamente diferentes.

Tienes razón al decir que ese artículo contiene la respuesta a esta pregunta, pero creo que la motivación es más formal que física. En realidad, el artículo que cité contiene una referencia a ese artículo, por lo que esta pregunta surgió por un descuido. Creo que en breve publicaré una respuesta explicando todo (también mi descuido).
@valerio92 sí, sería genial ver una resolución completa del problema, que es sorprendentemente complejo e interesante. ¡Gracias por mencionarlo en SE! Sin embargo, ¿crees que señalé correctamente la falla en el argumento de Callen? Encuentro que la razón por la que está mal es bastante obvia ahora, aunque no pretendo entender el alcance completo de los documentos que vinculó y cómo resolver el problema, pero pensé que esa no era la pregunta que hizo aquí, ¿verdad?
Creo que lo que dices es correcto, pero no es a lo que se refería el autor del artículo citado cuando escribió que "el argumento de Callen [...] no podía ser correcto ya que la condición de equilibrio se derivó de la primera ley, en lugar de la segunda ley". Y sí, aquí solo quiero discutir el argumento de Callen, no pretendo resolver el problema del pistón adiabático :-)
(Escribí que lo que dijiste es correcto, pero debo especificar que Callen no asume que T1=T2T1=T2, por lo que ese no puede ser el defecto de su argumento. Pero el resto de tu publicación es correcta, aunque en realidad no responde la pregunta.)
Después de una lectura más profunda del artículo de Curzon, estoy convencido de que no contiene la respuesta a mi pregunta y que su argumento es el mismo que el de Callen. Puede encontrar todos los detalles en la sección "actualizar" de mi pregunta. Perdón por tantos comentarios, prometo que este es el último!
@ valerio92 actualicé mi respuesta. La intención principal de la respuesta no ha cambiado, sin embargo, espero que esto ayude a responder a su pregunta.

Qué quiere decir

Luego se observó que el argumento de Callen, que fue repetido por Leff, no podía ser correcto ya que la condición de equilibrio se derivó de la primera ley, en lugar de la segunda ley.

Es esto. La primera ley es un enunciado de la conservación de la energía. Se mantiene incluso si el sistema no está en equilibrio termodinámico. Si el argumento fuera correcto implicaría que nunca vemos un desequilibrio de presión en la naturaleza, lo cual es falso.

¿Cuál es la falla en el argumento entonces? Es esto. El movimiento del pistón hacia su posición de equilibrio es irreversible, por lo que la ecuación d tu i = d W i = PAGS i d V i Es falso ( d W i siendo trabajo realizado sobre el sistema). En cambio tenemos la desigualdad d tu i = d W i PAGS i d V i con igualdad sólo para procesos reversibles. Si el proceso de equilibrio fuera reversible, entonces, como usted dice, "el principio de máxima entropía no es concluyente" porque cualquier movimiento del pistón deja la entropía total sin cambios.

Lo que muestra el argumento de Curzon es que, cuando el pistón se mueve hacia su posición de equilibrio, su incremento final de volumen se intercambia de manera reversible. Esto se puede ver porque él deriva d tu i = PAGS i d V i de la igualdad d S = 0 (sería falso de lo contrario). Esta igualdad de diferenciales se cumple solo cuando el pistón está en su posición de equilibrio, por lo que la segunda ley se ha utilizado de manera esencial.

Va mal cuando se aplica la conservación de la energía a la energía interna total. Aquí hay que sumar la energía cinética del pistón. Esto permite que las dos presiones sean diferentes, entonces el pistón oscilará alrededor de su posición de equilibrio. La disipación de esta energía conducirá a un aumento de la entropía; a la entropía máxima, el pistón se habrá asentado en la posición de equilibrio y las presiones se habrán igualado.

No estoy seguro de que esto sea realmente un defecto. Quiero decir, es un argumento en termodinámica de equilibrio. Aquí afirma que es incorrecto para un sistema fuera de equilibrio (verdadero) y que volverá a ser correcto cuando el sistema logre el equilibrio. O sea, que es válido para la termodinámica de equilibrio, ¿no?
@dmckee Si comparamos esto con, por ejemplo, un sistema aislado que consta de dos subsistemas que son un equilibrio térmico interno a diferentes temperaturas. Entonces podemos ver que la entropía total es máxima implica que las dos temperaturas deben ser iguales. Si no son iguales, entonces no tienes equilibrio térmico. Pero es posible tratar este caso usando métodos termodinámicos de equilibrio ya que los dos subsistemas están en equilibrio térmico interno. El caso de dos presiones diferentes debe considerarse de manera análoga.
La complicación es solo que la presión desigual nos deja con dos subsistemas con presiones oscilantes, pero aún puede tratar eso aproximadamente dentro del marco de la termodinámica de equilibrio. Al no tener en cuenta la situación física donde las presiones no son iguales, se están imponiendo las condiciones válidas en el estado final y eso oscurece el papel de la segunda ley para lograr el equilibrio entre las presiones.
También puede decir que el objetivo de la segunda ley es considerar cómo se alcanza el equilibrio, por lo que debe considerar una situación de no equilibrio. Que tengamos trucos disponibles para hacer eso dentro del marco de la termodinámica del equilibrio (dividir un sistema que no está en equilibrio en partes de modo que cada una esté en equilibrio interno y los cambios ocurran de forma casi estática) es algo bueno, pero deberíamos No te dejes llevar demasiado por los detalles del uso de estos trucos. Lo que importa es que exista un estado inicial bien definido y un estado final bien definido.
No creo que nada de lo que haya dicho aquí esté mal (y no es mi voto negativo), pero sí creo que representan razones filosóficas para preferir un argumento diferente en lugar de algún tipo de invalidación del enfoque de Callen. Claro, no se generaliza muy bien. Claro, no aprovecha una oportunidad pedagógica para presumir la segunda ley. ¿Y qué? Los análisis de casos especiales suelen ser una gran victoria para algunos casos y no buenos para otros.
Creo que este comentario es realmente interesante, pero desafortunadamente esto claramente no es lo que el autor del artículo quiere decir cuando dice que el argumento "no podría ser correcto ya que la condición de equilibrio se derivó de la primera ley, en lugar de la segunda ley". También creo que ese problema podría evitarse diciendo que la masa del pistón es tal que su energía es despreciable. Sé que esto no es realista, pero en termodinámica tales suposiciones se hacen todo el tiempo.
@valerio92 El problema es que las suposiciones ocultas habituales no se aplican y cuando se hacen estas suposiciones, como dU = 0, se supone implícitamente que el sistema ya tiene la entropía máxima, lo que dará como resultado que las presiones sean iguales. Entonces parece que la igualdad de presión no se sigue de la segunda ley porque cuando la segunda ley se invocó explícitamente no arrojó nada útil, mientras que la primera ley sí arrojó el resultado. Sin embargo, la razón por la cual invocar la segunda ley no produjo nada útil fue porque se aplica de manera redundante.

El argumento de Callen parece argumento del trabajo virtual. Si el sistema está en equilibrio, entonces un pequeño desplazamiento consistente con las restricciones existentes en el sistema debería terminar sin realizar ningún trabajo. Esto, por lo que puedo decir, es un argumento correcto. Sin embargo, quizás la objeción es que esto no cuenta como un argumento termodinámico, que debe hacer un uso explícito de la maximización de la entropía.

Empezamos con la forma diferencial de la entropía total S del sistema compuesto aislado de 1 & 2 .

d S t o t a yo = d S 1 + d S 2 = ( 1 / T 1 ) d tu 1 + ( PAGS 1 / T 1 ) d V 1 + ( 1 / T 2 ) d tu 2 + ( PAGS 2 / T 2 ) d V 2

Luego aplicamos que dado que el sistema compuesto está aislado, d tu t o t a yo = d V t o t a yo = 0 , asi que d tu 2 = d tu 1 y d V 2 = d V 1

d S t o t a yo = ( 1 / T 1 1 / T 2 ) d tu 1 + ( PAGS 1 / T 1 PAGS 2 / T 2 ) d V 1

La condición de equilibrio es que d S t o t a yo = 0 , por lo que cada término debe ser 0 .

( 1 / T 1 1 / T 2 ) d tu 1 = 0

( PAGS 1 / T 1 PAGS 2 / T 2 ) d V 1 = 0

Como el volumen interno no es fijo, d V 1 no es 0 , y por lo tanto ( PAGS 1 / T 1 PAGS 2 / T 2 ) debe ser 0 . Dado que el volumen no es fijo, es posible que tu 1 a cambiar (a través del trabajo de volumen), y por lo tanto d tu 1 no es 0 o. Por lo tanto ( 1 / T 1 1 / T 2 ) también debe ser 0 .

Combinando ambas ecuaciones (y suponiendo que ninguna T es cero) obtenemos dos condiciones de equilibrio:

T 1 = T 2 y PAGS 1 = PAGS 2

Nótese que la adiabaticidad de la pared no era necesaria para la deducción. De hecho es irrelevante. Si la pared es móvil, las temperaturas se igualarán en el equilibrio, por lo que podríamos decir que demostramos que ninguna pared es adiabática a menos que esté completamente aislada.

d S t o t a yo = 0 es la condición de equilibrio y su fórmula no incluye ni 'calor q 'ni' trabajo W ', solamente d tu 1 y d V 1 . Cualquier prueba que involucre q o W será menos fundamental que éste, ya que implica añadir postulados adicionales sobre la relación entre q y W y tu .

Además, tengo entendido ( como se dice en la respuesta de UtilityMaximiser) que la fórmula d W 1 , 2 = PAGS 2 , 1 d V 1 , 2 no siempre es válido. Sólo es válido si PAGS 1 , 2 está bien definido. Y la única situación en la que ambos PAGS 1 y PAGS 2 están bien definidos simultáneamente es si el sistema está en equilibrio.

Por lo tanto, al proponer d tu 1 , 2 = PAGS 2 , 1 d V 1 , 2 ya se está imponiendo que el sistema esté en equilibrio, por lo que cualquier otra condición de equilibrio se vuelve imposible de obtener.